Đến nội dung

Hình ảnh

VMF's Marathon Hình học Olympic

* * * * * 1 Bình chọn hình học

Lời giải halloffame, 02-01-2018 - 16:29

Lời giải bài toán 196. Ta chứng minh bài toán cho đường tròn $(K)$ tiếp xúc trong $(O),$ trường hợp tiếp xúc ngoài chứng minh tương tự. Ta thấy có thể bỏ đi điểm $B$ không cần thiết.

Bài toán 196'. $\Delta ADC$ vuông tại $D$ nội tiếp $(O),$ một đường tròn $(E)$ tiếp xúc trong $(O)$ ở $T.M,N \in (E)$ sao cho $MN \parallel AD$ và $MN=AD.P,R$ là trung điểm $MD,MC.$ Khi đó $P \in (ORT).$

Chứng minh. 

$M'$ đối xứng $M$ qua $T.$ Dựng điểm $I$ sao cho $OEMI$ là hình bình hành.

$OI$ cắt $(O),CD$ ở $K,L.J$ là hình chiếu $I$ lên $CD.$

Từ $OEMI$ là hình bình hành và $EM=ET$ ta suy ra được $IK=IM=JN,LK=LM$

Gọi $Q$ đối xứng $M$ qua $O$ thì $Q \in LM'.$ Ta có $LM'.LQ=LK.NJ=LK.KI=KO^2-OL^2=LC.LD \Rightarrow Q \in (M'CD).$

Qua phép vị tự tâm $M$ tỉ số $\frac{1}{2}$ ta có ngay đpcm.

[attachment=33194:Screen Shot 2018-01-02 at 1.29.42 AM.png]

Đi đến bài viết »


  • Please log in to reply
Chủ đề này có 434 trả lời

#241
moonkey01

moonkey01

    Hạ sĩ

  • Thành viên
  • 50 Bài viết

Bài toán 108 (Mở rộng ý a) bài toán 7 VMO 2017). Cho tam giác $ABC$ nội tiếp đường tròn $(O)$. Một đường tròn $(K)$ đi qua $B,C$. Trung trực $BC$ cắt $(K)$ tại $M,N$. $P$ là điểm thuộc $(K)$. $PM$ cắt $CA,AB$ tại $E,F$. $BE$ cắt $CF$ tại $L$. Chứng minh rằng $AL,PN$ và $BC$ đồng quy.

 

attachicon.gifFigure4244.png

 

Không rõ lời giải của em có nhầm lẫn gì không, nhưng em thấy lời giải khá ngắn.

 

Gọi $NP$ cắt $BC$ tại $T$, $EF$ cắt $BC$ tại $S$ thì $PN$ là phân giác $\angle BPC$ nên $(ST,BC)=-1$, từ đó dễ dàng có ngay $AL$ đi qua $T$.

 

Bài toán tiếp theo là của thầy Hùng, là bài toán 25 trong tập bài giảng của thầy tại đây: http://analgeomatica...t-chuyen-i.html. Nói ngoài lề một ít, đây là tập bài giảng làm em rất ấn tượng, một phần vì các bài sau rất khó, và mỗi lần sau một khoảng thời gian giải lại thì lại tìm thấy những điều thú vị.

 

Bài toán 109 (Sáng tác từ Serbia 2013 và Iran 2015): Cho tam giác $ABC$ nội tiếp $(O)$, $X$ là điểm trên cung nhỏ $BC$ sao cho nếu $E,F$ là hình chiếu của $X$ trên $IB,IC$ thì trung điểm $EF$ nằm trên trung trực $BC$. Gọi $J$ là tâm bàng tiếp góc $\angle BAC$ của tam giác $ABC$. Chứng minh rằng $XJ$ đi qua trung điểm cung lớn $BC$ của $(O)$.


Bài viết đã được chỉnh sửa nội dung bởi moonkey01: 10-01-2017 - 17:33


#242
manhtuan00

manhtuan00

    Trung sĩ

  • Thành viên
  • 108 Bài viết

$\boxed { Lời \ giải \ bài \ 109 }$ Bài toán trên tương đương với bài toán sau 

Cho $\triangle ABC$ với $I,J$ lần lượt là tâm nội tiếp và tâm bàng tiếp góc $\angle A$. $K$ là trung điểm cung lớn $BC$. $JK$ cắt $(O)$ tại $X$. $E,F$ là hình chiếu của $X$ lên $IC,IB$. Chứng minh rằng trung điểm $EF$ nằm trên trung trực $BC$

 

Chứng minh : Gọi $M,N, G$ lần lượt là tâm bàng tiếp góc $\angle B, \angle C$, trung điểm $BC$. Ta cần chứng minh $KG$ đi qua trung điểm $EF$, $JK$ cắt $BC$ tại $T$

Ta sẽ chứng minh $\frac{BF}{CE} = \frac{BM}{CN}$

Ta có : $\frac{BF}{CE} = \frac{BF}{BX} .\frac{BX}{CX} . \frac{CX}{CE} = \frac{cos \angle FBX}{cos \angle ECX} . \frac{sin \angle XCB}{sin \angle XBC} = \frac{sin \angle XBJ}{sin \angle XCJ} . \frac{sin \angle XCB}{sin \angle XBC}$

Gọi $U,V$ là trung điểm cung lớn $AC,AB$ khi đó 

$\frac{sin \angle XBJ}{sin \angle XCJ} . \frac{sin \angle XCB}{sin \angle XBC} = \frac{sin \angle XBJ}{sin \angle XCJ} . \frac{sin \angle XCB}{sin \angle XBC} $

$= \frac{sin \angle JKU}{sin \angle JKV} . \frac{sin \angle JKB}{sin \angle JKC} $
$= \frac{sin \angle KJM}{sin \angle KJN} . \frac{sin \angle JKB}{sin \angle JKC} $
$= \frac{JN}{JM} . \frac{sin \angle TKB}{sin \angle TKC} $
$= \frac{JN}{JM} . \frac{ TB}{TC} = \frac{JN}{JM} . \frac{ JM^2}{JN^2}= \frac{JM}{JN}$
Vậy ta có $\frac{BF}{CE}=  \frac{JM}{JN}$ nên theo bổ đề E.R.I.Q ta có trung điểm $BC,EF,MN$ thẳng hàng tức là trung điểm $EF$ nằm trên trung trực $BC$
 
$\boxed { Bài \ toán \ 110} $ (sách) Cho $\triangle ABC$ nội tiếp $(O)$ , $K,L$ là tâm bàng tiếp góc $\angle B, \angle C$. $(K)$ tiếp xúc $BA,BC$ tại $B_1,B_2$, tương tự ta có $C_1,C_2$. $B_1B_2$ cắt $C_1C_2$ tại $N$. Chứng minh rằng $AN \perp BC$


#243
moonkey01

moonkey01

    Hạ sĩ

  • Thành viên
  • 50 Bài viết

 

$\boxed { Bài \ toán \ 110} $ (sách) Cho $\triangle ABC$ nội tiếp $(O)$ , $K,L$ là tâm bàng tiếp góc $\angle B, \angle C$. $(K)$ tiếp xúc $BA,BC$ tại $B_1,B_2$, tương tự ta có $C_1,C_2$. $B_1B_2$ cắt $C_1C_2$ tại $N$. Chứng minh rằng $AN \perp BC$

 

 

Lời giải của em như sau. Em tính hết ra.

 

Không mất tính tổng quát, giả sử vị trí các điểm như hình vẽ. Đặt $BC=a, CA=b, AB=c$. Gọi đường cao $AH$ của tam giác $ABC$ cắt $CL,BK,B_1B_2,C_1C_2$ tại $X,Y,T,T'$. Ta có $\angle HT'C_2=\frac{\angle ACB}{2}=\angle ACL$ nên $CXC_1T'$ nội tiếp hay $AX.AT'=AC.AC_1=b(p-b)$. Tương tự thì $AY.AT=AB.AB'=c(p-c)$ nên $\frac{AX}{AY}.\frac{AT'}{AT}=\frac{b(p-b)}{c(p-c)}$. Ta tính được $BH=AB.cos\angle ABC=\frac{c(c^2+a^2-b^2)}{2ca}=\frac{c^2+a^2-b^2}{2a}$, mặt khác theo tính chất đường phân giác thì $\frac{AY}{AB}=\frac{YH}{BH}=\frac{AH}{AB+BH}$ nên $AY=\frac{AB.AH}{AB+BH}=\frac{c.AH}{c+\frac{c^2+a^2-b^2}{2a}}=\frac{2ac.AH}{(c+a-b)(c+a+b)}=\frac{ac.AH}{2p(p-b)}$, tương tự thì $AX=\frac{ab.AH}{2p(p-c)}$. Do đó $\frac{AX}{AY}=\frac{b(p-b)}{c(p-c)}=\frac{AX}{AY}.\frac{AT'}{AT}$, dẫn đến $AT=AT'$ hay $B_1B_2,C_1C_2$ cắt nhau trên $AH$. Vậy $AN\perp BC$ $\blacksquare$

 

Và em muốn nhờ thầy Hùng đề nghị bài toán mới :)

Hình gửi kèm

  • Ảnh chụp Màn hình 2017-01-10 lúc 23.52.43.png

Bài viết đã được chỉnh sửa nội dung bởi moonkey01: 11-01-2017 - 00:23


#244
Ngockhanh99k48

Ngockhanh99k48

    Trung sĩ

  • Thành viên
  • 127 Bài viết

Một cách khác cho bài toán 110:

Gọi $J$ là tâm bàng tiếp góc $A$ của $\triangle ABC$. $A_1$ là hình chiếu của $J$ trên $BC$. $B_3, C_3$ là hình chiếu của $K, L$ trên $AC, AB$.

Theo tính chất của tâm bàng tiếp ta có $A_1C=AC_3$, $A_1B=AB_3$ và $CB_2=CB_3=BC_2=BC_3$. Suy ra: $A_1B_2=A_1C+CB_2=AC_3+BC_3=AB$, $A_1C_2=AC$.

Ta có $(CA^2 - CB_2^2) + (JB_2^2 - JC_2^2) + (BC_2^2 - BA^2) =  (CA^2 - BA^2) + (A_1B_2^2 - A_1C_2^2) = 0$. Như vậy: Đường thẳng qua $C$ vuông góc $AB_2$, đường thẳng qua $B$ vuông góc $AC_2$ và đường thẳng qua $J$ vuông góc $B_2C_2$ đồng quy theo định lí Carnot. Do đó ta suy ra đường thẳng qua $C_2$ vuông góc với $JB$, đường thẳng qua $B_2$ vuông góc với $JC$ và đường thẳng qua $A$ vuông góc $BC$ đồng quy. Tức là $C_2C_3, B_2B_3$ và đường cao $AH$ của tam giác $ABC$ đồng quy. Mặt khác $C_2C_3, B_2B_3$ là các đường cao của $\triangle NB_2C_2$ nên ta có $A, N, H$ thẳng hàng. 

P/s: Xin giải lại bài toán 109 theo cách tam giác đồng dạng: cùng ý tưởng với manhtuan00

$IB$ cắt $JC$ tại $M$, $IC$ cắt $JB$ tại $N$. Ta chứng minh $BC, EF, MN$ có 3 trung điểm thẳng hàng, hay $\frac{BE}{BM}=\frac{CF}{CN}$. Ta có $XK$ là phân giác $\widehat{BXC}$ nên $\widehat{KXB}=\widehat{KXC}=\widehat{BJC}$. Suy ra $\widehat{XBJ}= \widehat{XJC}$ hay $\triangle XBJ \sim \triangle XJC$. Gọi $Y, Z$ là hình chiếu của $X$ trên $JB, JC$ thì $\frac{XY}{XZ}=\frac{JB}{JC}$. Mặt khác $BEXY$ và $CFXZ$ là các hình chữ nhật nên $\frac{JB}{JC}=\frac{BE}{CF}$. Ta lại có $\triangle JBM \sim \triangle JCN (g.g)$ nên $\frac{BE}{CF}= \frac{BM}{CN}$ ta có đpcm.


Bài viết đã được chỉnh sửa nội dung bởi Ngockhanh99k48: 11-01-2017 - 10:02


#245
quanghung86

quanghung86

    Thiếu úy

  • Điều hành viên
  • 632 Bài viết

Cám ơn Hoàng và Khánh, một lời giải khác cho bài toán 107 của thầy Hà ở đây, rất thú vị và ngắn gọn

 

https://www.artofpro...c6t48f6h1366405

 

Thầy giúp Hoàng đề nghị bài toán sau

 

Bài toán 111. Cho tam giác $ABC$ nội tiếp đường tròn $(O)$. Đường tròn tâm nội tiếp $I$. Đường tròn $(K)$ tiếp xúc $CA,AB$ tại $E,F$ và tiếp xúc trong $(O)$. $P$ là trung điểm cung $BC$ chứa $A$ của $(O)$. $PI$ cắt $BC$ tại $D$. $Q$ là trung điểm cung nhỏ $AP$ của $(O)$. $R$ là trung điểm cung nhỏ $EF$ của $(K)$. $M$ là trung điểm $ID$. Chứng minh rằng $MQ\parallel DR$.

 

Figure4301.png


Bài viết đã được chỉnh sửa nội dung bởi quanghung86: 11-01-2017 - 10:27


#246
quanghung86

quanghung86

    Thiếu úy

  • Điều hành viên
  • 632 Bài viết

Mình đưa ra tổng quát bài toán 109 và lời giải của mình, nguồn gốc bài 109 là bài số 6 chọn đội Iran 2014 chứ không phải 2015

 

https://www.artofpro...h590555p3497371

 

Bài toán 109'. Cho tam giác $ABC$ nội tiếp đường tròn $(O)$ với tâm nội tiếp $I$ và tâm bàng tiếp góc $A$ là $J$. $P$ là trung điểm cung $BC$ chứa $A$ của $(O)$. $X$ thuộc $JP$. $E,F$ là hình chiếu của $X$ lên $IB,IC$. Chứng minh rằng $OP$ chia đôi $EF$.

 

Figure4302.png

 

Lời giải. Gọi $K,L$ là tâm bàng tiếp góc $B,C$ của $ABC$ thì $P$ là trung điểm $KL$. Gọi $XF,XE$ cắt $JC,JB$ tại $QR$. Dễ thấy do $XF\parallel JB$ và $XE\parallel JC$ nên $QR\parallel BC$. Từ đó $\frac{FB}{FK}=\frac{QJ}{QK}=\frac{RJ}{RL}=\frac{EC}{EL}$ nên theo bổ đề E.R.I.Q thì trung điểm của $KL,EF,BC$ thẳng hàng. Ta hoàn thành chứng minh.


Bài viết đã được chỉnh sửa nội dung bởi quanghung86: 11-01-2017 - 11:03
Sửa số thứ tự


#247
Ngockhanh99k48

Ngockhanh99k48

    Trung sĩ

  • Thành viên
  • 127 Bài viết

$\boxed{\text{Lời giải bài 111}}$
$AK$ cắt $(K)$ tại điểm thứ hai $N$. $(K)$ tiếp xúc $(O)$ tại $X$.
Ta có $\triangle XBF \cap I \stackrel{+}{\sim} \triangle XIE \cap C (g.g) $ . Suy ra $XI$ là phân giác $\widehat{BXC}$ nên $X$ thuộc $PI$.
$\overline{AR}.\overline{AN}=AF^2=\overline{AI}.\overline{AK}$ nên $(AIRN)=-1$, mặt khác $XR \perp XN$ nên XR là phân giác trong $\widehat{AXP}$ hay $X$ thuộc $QR$.
Phép vị tự tâm $X$ biến $(K)$ thành $(O)$ đã biến $R$ thành $Q$ nên $AK \parallel OQ$. Mặt khác $OQ$ là trung trực $AP$ và $AK \perp AP$ nên $OQ$ chia đôi $PI$ tại $Y$. Ta có $\triangle XBD \sim \triangle XPC$ và $\triangle XBI \sim \triangle XIC$ nên $XD.XP=XB.XC=XI^2$, tức là $\frac{XD}{XI}=\frac{XI}{XP}$. Do $M, Y$ thứ tự là trung điểm $DI, IP$ nên $\frac{XD}{DM}=\frac{XI}{IY}=\frac{XR}{RQ}$ do $RI \parallel QY$. Theo Thales ta có $RD \parallel MQ$.

$\boxed{\text{Bài toán 112}}$ Cho $\triangle ABC$ nội tiếp $(O)$ và điểm Lemoine $L$. $AL$ cắt $(O)$ tại điểm thứ hai $X$. Đường tròn $(K)$ qua $L$ tiếp xúc trong $(O)$ tại $X$. Đường thẳng qua $K$ vuông góc $BC$ cắt $OL$ tại $T$. Chứng minh $\overrightarrow{OT}= \frac{1}{4} \overrightarrow{OL}$


Bài viết đã được chỉnh sửa nội dung bởi Ngockhanh99k48: 11-01-2017 - 20:34


#248
quanghung86

quanghung86

    Thiếu úy

  • Điều hành viên
  • 632 Bài viết

Cảm ơn Khánh về lời giải thú vị khác đáp án, đây là hình vẽ cho lời giải của em. Đáp án gốc thầy sẽ gõ và post vào hôm sau.

 

Figure4248.png



#249
quanghung86

quanghung86

    Thiếu úy

  • Điều hành viên
  • 632 Bài viết
Lời giải bài toán 112. Gọi tiếp tuyến qua $B,C$ của $(O)$ cắt nhau tại $S$. $TK$ tại $J$. Do $(K)$ tiếp xúc $(O)$ nên $KL\parallel AO$. Từ đó $\frac{JS}{XS}=\frac{KO}{XO}=\frac{LA}{XA}$ suy ra $\frac{JS}{LS}=\frac{XS}{LS}:\frac{XA}{LA}$. Ta thấy các hàng điểu hòa cơ bản $(AX,SD)=(AD,SL)=-1$ do dó $(AS,XD)=(AS,DL)=2$ suy ra $\frac{XA}{XS}:\frac{DA}{DS}=\frac{DA}{DS}:\frac{LA}{LS}=2$ từ đó dễ thấy $\frac{XS}{LS}:\frac{XA}{LA}=\frac{1}{4}$ hay $\frac{OT}{OL}=\frac{JS}{JL}=\frac{1}{4}$. Ta hoàn thành chứng minh.

IMG_0997.PNG

Bài toán 113. Cho tam giác $ABC$ và $P$ bất kỳ. $D,E,F$ là hình chiếu của $P$ lên $BC,CA,AB$. $(DEF)$ cắt $BC$ tại $M$ khác $D$. $DP$ cắt $EF$ tại $L$. $AL$ cắt $BC$ tại $K$. Các đường thẳng qua $A,M$ lần lượt vuông góc với $BC,PK$ cắt nhau tại $X$. Chứng minh rằng $XK\parallel EF.$

IMG_0998.PNG

#250
NHN

NHN

    Hạ sĩ

  • Thành viên
  • 84 Bài viết

Lời giải bài toán 113. Gọi điểm liên hợp dăng giác của $P$ là $Q$. Giao điểm của $AQ$ và $BC$ là $T$ thì tam giác $DLE$ đồng dạng tam giác $CTQ$, nên $\widehat{CTQ}=\widehat{DLE}$ vậy $\frac{AQ}{QT}=\frac{\sin(\widehat{ACQ}).\sin(\widehat{ATC})}{\sin(\widehat{QCT}).\sin(\widehat{QAC})}=\frac{\sin(\widehat{PED}).\sin(\widehat{DLE})}{\sin(\widehat{LDE}).\sin(\widehat{LEP})}=\frac{PD}{PL}$ ta có $QM$ vuông $BC$, gọi hình chiếu của $A$ lên $BC$ là $H$ suy ra $\frac{HM}{MT}=\frac{AQ}{QT}$ mà do ta thầy tam giác $PKD$ đồng dạng tam giác $MXH$ vậy tam giác $MXT$ đồng dạng tam giác $PKL$ vậy $TX$ vuông $AL$, vậy $X$ là trực tâm tam giác $ATK$ vậy $KX$ vuông $AT$ mà ta có tính chất là $AQ$ vuông $EF$ vây $EF$ song song $KX$.

 

 

Bài toán 114. Cho tam giác $ABC$, hình chiếu của $A,B,C$ lên $BC,AC,AB$ là $D,E,F$. Gọi hình chiếu của $D$ lên $AB,AC$ là $X,Y$. Gọi giao của $XY,EF$ và $BC$ là $K,L$. Đường thẳng vuông với $LA$ tại $A$ cắt $BC$ tại $R$. Giao của $KA$ và $(ABC)$ là $Z$ chứng minh $RZ=RA$.

Hình gửi kèm

  • Capture.JPG

Bài viết đã được chỉnh sửa nội dung bởi NHN: 13-01-2017 - 19:39


#251
Ngockhanh99k48

Ngockhanh99k48

    Trung sĩ

  • Thành viên
  • 127 Bài viết

Lời giải bài 114. Gọi $O$ là tâm ngoại tiếp $\triangle ABC$.Ta có $AD^2=AX.AB=AY.AC$ nên $X, Y, B, C$ đồng viên. Mặt khác dễ thấy $BC$ tiếp xúc $(AD)$ nên $KD^2=KX.KY=KB.KC=KA.KZ$ và $\widehat{ADK}=90^{\circ}$ nên $DZ \perp AK$ hay $Z \in (AD)$. $LA$ cắt $(O)$ tại $P$. Gọi $H$ là trực tâm $\triangle ABC$, $M$ là trung điểm $BC$. Khi đó ta có $H, P, M$ thẳng hàng và $HM \perp LA$, suy ra $AR \perp HM$. $OM \cap AR = Q$ thì $AHMQ$ là hình bình hành, tuy nhiên $\overrightarrow{AH}=2\overrightarrow{OM}$ nên $O$ là trung điểm $MQ$. Theo Thales ta có $OR$ chia đôi $AD$. Mặt khác trung điểm $AD$, $O$ thuộc trung trực $AZ$ nên $R$ cũng huộc trung trực $AZ$. Ta có đpcm.

 

Figure4249.png

 

Bài toán 115. (Thầy Nguyễn Minh Hà) Cho tam giác $ABC$ nội tiếp đường tròn $O$ và điểm Lemoine $L$. $AL$ cắt $(O)$ tại điểm thứ hai $X$. $(K)$ là đường tròn qua $L$ tiếp xúc trong $(O)$ tại $X$ cắt $BC$ tại $Y, Z$. Điểm $S$ chia đoạn $OL$ theo tỉ số $\frac{-1}{3}$. $YL$ cắt $(S, SY)$ tại điểm thứ hai $E$. Chứng minh $(XYE)$ chia đôi $AL$.   


Bài viết đã được chỉnh sửa nội dung bởi quanghung86: 13-01-2017 - 18:51


#252
quanghung86

quanghung86

    Thiếu úy

  • Điều hành viên
  • 632 Bài viết

Figure4250.png

 

Lời giải khác cho bài toán 113. Gọi $Q$ đẳng giác với $P$ trong tam giác $ABC$ thì $M$ là hình chiếu của $Q$ lên $BC$. Dễ thấy $AQ\perp EF$ nên ta sẽ chứng minh $AQ\perp XK$ thì suy ra $EF\parallel XK$, thật vậy. Gọi $AP$ cắt $(O)$ ngoại tiếp tam giác $ABC$ tại $Y$ khác $A$. Gọi đường thẳng qua $L$ song song $BC$ cắt $CA,AB$ tại $Z,T$. Ta thấy $\angle PTZ=\angle PFE=\angle PAE=\angle YBC$ và $\angle PZT=\angle PEF=\angle PAB=\angle YCB$ do đó hai tam giác $PZT$ và $YCB$ đồng dạng có đường cao tương ứng là $PL,YK$. Từ đó $\frac{LZ}{LT}=\frac{KC}{KB}$ nên $A,L,K$ thẳng hàng. Từ đó theo bài toán Tuần 1 tháng 4/2016 ta có $AQ\perp XK$. Ta hoàn thành chứng minh.


Bài viết đã được chỉnh sửa nội dung bởi quanghung86: 13-01-2017 - 19:28


#253
baopbc

baopbc

    Himura Kenshin

  • Thành viên nổi bật 2016
  • 410 Bài viết

Lời giải bài toán 115. Tiếp tuyến tại $A$ của $(O)$ cắt $BC$ tại $G$. Do tứ giác $ABXC$ điều hòa nên $GX$ tiếp xúc $(O)$, mặt khác do $(K)$ tiếp xúc $(O)$ tại $X$ nên $GX$ tiếp xúc $(K)$. Theo bài toán 112 thì $KS\perp BC$ nên $SY=SZ$. Do đó $GA^2=GX^2=GY\cdot GZ=\mathcal{P}_{G/(S)}=GS^2-SY^2$.

 

Theo định lý Cosin thì $GS^2=GO^2+OS^2-2OS.OG.\cos (\widehat{SOG})=GO^2+OS^2-\tfrac{1}{2}OL.OG.\cos (\widehat{SOG})$.

Gọi $T$ là giao điểm của $OG$ với $AX$. Do $OG\perp AX$ nên $OL.\cos(\widehat{SOG})=OT$, suy ra

$GS^2=GO^2+OS^2-\tfrac{1}{2}OG.OT=GO^2+OS^2-\tfrac{1}{2}R^2$

$\Rightarrow GA^2=GO^2-\tfrac{1}{2}R^2+OS^2-SY^2\Rightarrow SY^2-OS^2=\tfrac{1}{2}R^2$.

 

Gọi $F$ là giao điểm của $(XYE)$ với $AX$.

Ta có $LF\cdot LX=LY\cdot LE=SY^2-LS^2=SY^2-\tfrac{9}{16}LO^2=SY^2-OS^2-\tfrac{1}{2}OL^2=\tfrac{1}{2}(R^2-LO^2)=\tfrac{1}{2}LA\cdot LX$

$\Rightarrow F$ là trung điểm $AL$.$\square$


Bài viết đã được chỉnh sửa nội dung bởi baopbc: 13-01-2017 - 20:27


#254
quanghung86

quanghung86

    Thiếu úy

  • Điều hành viên
  • 632 Bài viết

Cám ơn Bảo, rất vui khi thấy sự trở lại của em. Theo đề nghị của em, thầy đề nghị một bài mới như sau

 

Bài toán 116. Cho tam giác $ABC$ nội tiếp đường tròn $(O)$ với tâm nội tiếp $I$. Đường tròn $(K)$ tiếp xúc $CA,AB$ tại $E,F$ và tiếp xúc trong $(O)$ tại $D$. $AI$ cắt $(O)$ tại $P$ khác $A$. $PD$ cắt $(K)$ tại $Q$ khác $D$. $DI$ cắt $BC$ tại $R$. Chứng minh rằng $RO$ đi qua tâm đương tròn Euler tam giác $QEF$.

 

Figure4251.png

 

@Trong topic này những bài nào mình post mà tự sáng tác mình sẽ không ghi gì cả, còn những bài không phải của mình mình sẽ ghi rõ.



#255
baopbc

baopbc

    Himura Kenshin

  • Thành viên nổi bật 2016
  • 410 Bài viết

Lời giải bài 116.

Bổ đề. Cho tam giác $ABC$ nội tiếp đường tròn $O$, trực tâm $H$, $M$ là trung điểm $BC$. Đường cao $BE$, $CF$. $MH$ cắt $EF$ và $(O)$ lần lượt tại $R$, $K$. $KA$ cắt $MO$ tại $P$. Đường thẳng qua $P$ vuông góc với $EF$ cắt $KO$ tại $Q$. Khi đó $QR$ đi qua tâm đường tròn Euler của tam giác $ABC$.

 

Chứng minh. Do $AH\parallel PM$ nên $\tfrac{MH}{MK}=\tfrac{PA}{PK}=\tfrac{QO}{QK}$. Mặt khác do $(KHRM)=-1$ nên $\tfrac{MH}{MK}=\tfrac{RH}{RK}$. Từ đó theo định lí Menelaus $RQ$ đi qua trung điểm $OH$ hay $QR$ đi qua tâm đường tròn Euler của tam giác $ABC$. Bổ đề được chứng minh.

 

Quay lại bài toán.

Ta có $\widehat{QFB}=\widehat{QEF}=180^\circ-\widehat{QDF}=90^\circ-\tfrac{1}{2}\widehat{ABC}$ (do $DF$ đi qua điểm chính giữa cung $AB$ không chứa $C$ của $\triangle ABC$). Gọi $M$ là giao điểm của $QF$ với $BC$ thì $\triangle FBM$ cân tại $B$ suy ra $IF=IM=IE$ suy ra $I$ là tâm đường tròn ngoại tiếp tam giác $FME$ kéo theo $\widehat{EMF}=90^\circ$. Xác định điểm $N$ tương tự thì $\widehat{ENF}=90^\circ$. Do $\widehat{QDI}=90^\circ$ nên $DI$ đi qua điểm $Q'$ đối xứng với $Q$ qua $K$. Do đó $DI$ đi qua trực tâm tam giác $QEF$. Từ đó theo bổ đề ta có điều phải chứng minh.



#256
quanghung86

quanghung86

    Thiếu úy

  • Điều hành viên
  • 632 Bài viết

Cám ơn Bảo vì lời giải rất nhanh và đẹp, theo đề nghị của em thầy để xuất tiếp bài sau

 

Bài toán 117. Cho lục giác lồi $ABCDEF$ thỏa mãn $AB=CD=EF$, $BC=DE=FA$ và $\angle A+\angle B=\angle C+\angle D=\angle E+\angle F$. Chứng minh rằng $\angle A=\angle C=\angle E$.

 

Bài toán 114 của bạn NHN có nhiều phát triển thú vị, mọi người hãy cùng quan tâm.


Bài viết đã được chỉnh sửa nội dung bởi quanghung86: 13-01-2017 - 22:13
Sửa lại đề


#257
halloffame

halloffame

    Thiếu úy

  • Điều hành viên OLYMPIC
  • 522 Bài viết

Lời giải bài toán 117.

Gọi $AF,BC,DE$ lần lượt cắt $BC,DE,AF$ tại $X,Z,Y.$ Xét $\widehat{BAF},\widehat{CBA}$ là hai góc ngoài của $\Delta XAB$ ta có:

$\widehat{X}+180^0=\widehat{BAF}+\widehat{CBA}.$ Tương tự $\widehat{Z}+180^0=\widehat{BCD}+\widehat{CDE} \Rightarrow \widehat{X}=\widehat{Z}.$

Tương tự $\widehat{X}=\widehat{Y} \Rightarrow \Delta XYZ$ đều. Gọi $O$ là điểm sao cho $AO,FO$ lần lượt song song $BC,DE.$

Khi đó $\widehat{AFO}=\widehat{Y}=60^0=\widehat{X}=\widehat{FAO} \Rightarrow \Delta AOF$ đều $\Rightarrow AO=AF=BC.$

Vậy $AO=BC,AO \parallel BC \Rightarrow AOCB$ là hình bình hành $\Rightarrow \widehat{CBA}=\widehat{COA}$ và $CO=AB=CD.$

Tương tự, $\widehat{FED}=\widehat{DOF}$ và $OD=DC \Rightarrow \Delta OCD$ đều $\Rightarrow \widehat{COD}=\widehat{AOF}=60^0.$

Do đó, $\widehat{CBA}+\widehat{DEF}=\widehat{COA}+\widehat{FOD}=240^0.$

Tương tự, $240^0=\widehat{BCD}+\widehat{AFE}=\widehat{FEY}+\widehat{Y}+\widehat{CDZ}+\widehat{Z} \Rightarrow \widehat{FEY}+\widehat{CDZ}=120^0.$

$\Rightarrow \widehat{CDE}+\widehat{DEF}=240^0=\widehat{CBA}+\widehat{DEF}\Rightarrow \widehat{CBA}=\widehat{CDE}.$

Làm tương tự với các góc còn lại, ta suy ra đpcm.

Spoiler


Sự học như con thuyền ngược dòng nước, không tiến ắt phải lùi.


#258
quanghung86

quanghung86

    Thiếu úy

  • Điều hành viên
  • 632 Bài viết

Cám ơn Quân đã đóng góp lời giải, bài này thầy mở rộng từ bài IMO 2005, xin đề nghị bài tiếp như sau

 

Bài toán 118. Cho tam giác $ABC$ nhọn trực tâm $H$, tâm ngoại tiếp $O$, $G$ là trọng tâm của tam giác $HBC$. $K$ thuộc đoạn $AH$ sao cho $\angle BKC=90^\circ$. $KG$ cắt trung trực $BC$ tại $L$. Các điểm $P,Q$ thuộc đoạn $BC$ sao cho $LP\parallel OB,LQ\parallel OC$. Các điểm $E,F$ thuộc đường thẳng $HC,HB$ sao cho $PE,QF$ cùng vuông góc với $BC$. Chứng minh rằng $EF=PE+QF$.


Bài viết đã được chỉnh sửa nội dung bởi halloffame: 30-12-2017 - 14:18


#259
quanghung86

quanghung86

    Thiếu úy

  • Điều hành viên
  • 632 Bài viết

Figure3169.png

 

Đáp án bài toán 117. Gọi $BC,DE,AF$ cắt nhau tạo thành tam giác $MNP$. Vì tổng các góc của lục giác là $720^\circ$ nên $\angle A+\angle B=\angle C+\angle D=\angle E+\angle F=240^\circ$. Vì vậy, ta dễ thấy tam giác $MNP$ đều. Dựng tam giác đều $DEO$ với $O$ nằm trong lục giác. Ta thấy $EOAF$ và $DOBC$ đều là hình bình hành, nên $AB=EF=OA=CD=OB$, vậy $OAB$ là tam giác đều. Do đó, $\angle AFE+\angle BCD=\angle AOE+\angle BOD=360^\circ-60^\circ-60^\circ=240^\circ$. Nhưng do $\angle EDC+\angle BCD=240^\circ$. Từ đó suy ra $\angle AFE=\angle EDC$. Tương tự, $\angle EDC=\angle CBA$. Từ đó  $\angle B=\angle D=\angle F$. Ta có điều phải chứng minh.


Bài viết đã được chỉnh sửa nội dung bởi quanghung86: 14-01-2017 - 10:49


#260
baopbc

baopbc

    Himura Kenshin

  • Thành viên nổi bật 2016
  • 410 Bài viết

Lời giải bài toán 118.

Gọi $D$ là đối xứng của $L$ qua $BC$ thì $DP\parallel OC$, $DQ\parallel OB$.

Ta có $\tfrac{MD}{MO}=\tfrac{ML}{MO}=\tfrac{HK}{HA}$. Từ đó $\tfrac{MP}{MC}=\tfrac{MQ}{MB}=\tfrac{HK}{HA}$

Gọi $X$, $Y$, $S$, $T$ lần lượt là hình chiếu của $K$, $H$ lên $CA$, $AB$. 

Do $\triangle AKY\sim \triangle CPE$ và $\tfrac{YA}{YT}=\tfrac{PC}{PM}$ nên $\triangle CME\sim \triangle ATK$

Từ đó $\widehat{EMC}=\widehat{KTY}$ nên $\triangle EMP\sim \triangle TKY$.

Tương tự $\triangle FMQ\sim \triangle KSX$.

Do tứ giác $TKSC$ nội tiếp nên $\widehat{KSX}=\widehat{YKT}$ suy ra $\triangle YKT\sim \triangle XSK$

Từ đó $\triangle MPE\sim \triangle FQM$ kéo theo $MP^2=PE\cdot QF\Rightarrow PQ^2=4PE\cdot QF$

$\Rightarrow EF^2=(QF-PE)^2+PQ^2=(QF-PE)^2+4PE\cdot QF=(QF+PE)^2\Rightarrow EF=QF+PE$.  $\square$







Được gắn nhãn với một hoặc nhiều trong số những từ khóa sau: hình học

0 người đang xem chủ đề

0 thành viên, 0 khách, 0 thành viên ẩn danh